LSAT and Law School Admissions Forum

Get expert LSAT preparation and law school admissions advice from PowerScore Test Preparation.

 mpoulson
  • Posts: 148
  • Joined: Mar 25, 2016
|
#23189
Team,

This was very insightful. I think I was in the ball park before and now I think I've found my seat. Thank you. Until next time.

- Micah
 cmorris32
  • Posts: 92
  • Joined: May 05, 2020
|
#75711
Hi PowerScore!

Is it correct to rationalize that answer choice D is correct for these reasons:

The conclusion to the politician's statement is that the bill should be adopted. According to the Mechanistic Approach, any new element in the conclusion will appear in the correct answer. Since the rest of the stimulus does not discuss the bill specifically being adopted, the correct answer choice must address the bill being adopted. Answer choice D is the only answer choice that addresses this, so it must be the correct answer?

Thank you!
Caroline :-D
 Adam Tyson
PowerScore Staff
  • PowerScore Staff
  • Posts: 5153
  • Joined: Apr 14, 2011
|
#76403
That's a perfect use of the Mechanistic Approach, Caroline! Excellent work!
 g_lawyered
  • Posts: 211
  • Joined: Sep 14, 2020
|
#89980
Hi P.S.,
The explanations posted have been helpful but I have remaining questions. Does the last sentence in the stimulus have conditional reasoning that we need to follow? Can someone please confirm if my reasoning is correct or incorrect?
I picked correct answer D using my reasoning:
Conc- 1st sentence
Premise 1: uses new concepts- Distracts driver and public safety
Premise 2: I wrote out conditional reasoning like this:
If illegal to use care phones while driving :arrow: people will stop using car phones

From this I predicted the assumption is: Car phones are bad for public safety.
This lead me to pick answer choice D. I wrote out conditional reasoning for answer choice D as:
Law reduced threat to Public safety :arrow: adopt law

Because this answer choice matched my prediction I chose it. If I write out conditional reasoning in premises or answer choices, do I need to write out the contrapositive? In this problem, I didn't and I was able to pick the correct answer.

Thanks in advance
 Rachael Wilkenfeld
PowerScore Staff
  • PowerScore Staff
  • Posts: 1358
  • Joined: Dec 15, 2011
|
#90139
Hi GGIBA003,

You are correct that there's conditional reasoning in the last sentence, but it's not a critical aspect of the argument. You certainly can diagram it in the way you did, but personally, I would not. The argument is understandable without the conditional diagram, and the critical aspect of the answer choice is addressing the jump from "this will help public safety" to "this is something that should be adopted."

Hope that helps!

Get the most out of your LSAT Prep Plus subscription.

Analyze and track your performance with our Testing and Analytics Package.